Quantcast
  • Register
PhysicsOverflow is a next-generation academic platform for physicists and astronomers, including a community peer review system and a postgraduate-level discussion forum analogous to MathOverflow.

Welcome to PhysicsOverflow! PhysicsOverflow is an open platform for community peer review and graduate-level Physics discussion.

Please help promote PhysicsOverflow ads elsewhere if you like it.

News

PO is now at the Physics Department of Bielefeld University!

New printer friendly PO pages!

Migration to Bielefeld University was successful!

Please vote for this year's PhysicsOverflow ads!

Please do help out in categorising submissions. Submit a paper to PhysicsOverflow!

... see more

Tools for paper authors

Submit paper
Claim Paper Authorship

Tools for SE users

Search User
Reclaim SE Account
Request Account Merger
Nativise imported posts
Claim post (deleted users)
Import SE post

Users whose questions have been imported from Physics Stack Exchange, Theoretical Physics Stack Exchange, or any other Stack Exchange site are kindly requested to reclaim their account and not to register as a new user.

Public \(\beta\) tools

Report a bug with a feature
Request a new functionality
404 page design
Send feedback

Attributions

(propose a free ad)

Site Statistics

205 submissions , 163 unreviewed
5,047 questions , 2,200 unanswered
5,345 answers , 22,709 comments
1,470 users with positive rep
816 active unimported users
More ...

  Possible research implications of proof of John Cardy's a-theorem in QFT

+ 12 like - 0 dislike
1098 views

According to this recent article in Nature magazine, John Cardy's a-theorem may have found a proof.

Question:

What would the possible implications be in relation to further research in QFT?
Specifically, what types of qft's would now be studied more closely?

This post has been migrated from (A51.SE)
asked Nov 16, 2011 in Theoretical Physics by UGPhysics (155 points) [ no revision ]
More background?

This post has been migrated from (A51.SE)
http://en.wikipedia.org/wiki/A-theorem

This post has been migrated from (A51.SE)
I added the citation tag, since this seems eligible for this http://meta.theoreticalphysics.stackexchange.com/questions/189/upcoming-journal-editions-read-em-and-ask-on-the-stack-and-get-prizes

This post has been migrated from (A51.SE)

2 Answers

+ 10 like - 0 dislike

The possible applications I can think of are in determining the phases of various QFTs. There are tons of applications like that, here are some ideas:

-- If the solutions to 't Hooft's conditions are too complicated (entail too many fermions such that their contribution to the IR values of $a$ is greater that $a$ in the UV) there must be symmetry breaking, because then we can match the anomalies in other ways (not just massless fermions).

-- If the broken symmetry group were too large there would be too many Nambu-Goldstone bosons and one would have to conclude that the symmetry is (at least partially) unbroken.

-- Many other applications of this type for theories which are strongly coupled. One could determine the right candidates for the IR physics and so on. See for instance the very recent http://arxiv.org/abs/1111.3402

There is a viable conjecture in three dimensions, by Myers-Sinha. It involves the entanglement entropy across an S^1. It has already been tested in many perturbative models and also in N=2 SUSY gauge theories in three dimensions. I sincerely believe it is correct. Also a similar entanglement entropy in two dimensions gives the central charge $c$ and a similar entanglement entropy in four dimensions gives precisely the $a$-anomaly. So there seems to be a universal story around the entanglement entropy which has not been uncovered yet.

This post has been migrated from (A51.SE)
answered Nov 16, 2011 by Zohar Ko (650 points) [ no revision ]
"...I happen to have Mr. McLuhan right here"

This post has been migrated from (A51.SE)
Still on my watch list.... Ugh.

This post has been migrated from (A51.SE)
+ 9 like - 0 dislike

The most straightforward use of the $a$-theorem is to determine what kinds of spontaneous symmetry breaking are possible. For example, in the usual QCD with three light flavors, at high energy one has a theory of fermions and gauge fields and at low energy one has a theory of pions. If you tried the same thing with a different, large enough of number of fermions you would get a violation of $a$ theorem, since the number of pions is essentially quadratic in the number of fermions. Therefore there can be no spontaneous symmetry breaking with a sufficiently large number of fermions. For QCD, we knew this already by other arguments, but one could imagine applying this argument to a theory where we had no real control over anything.

A proof in three dimensions would be great, since thats where a lot of hard condensed matter problems live. Actually, who needs a proof, the right conjecture would be great. But its a little hard to see how you get that since the arguments relies completely on anomalies.

Added This is just what I (mis)remember from hearing Zohar Komargodski's "On renormalization groups flows in diverse dimensions" which is available here

This post has been migrated from (A51.SE)
answered Nov 16, 2011 by BebopButUnsteady (330 points) [ no revision ]
+1 For the [PIRSA link](http://pirsa.org/11110115).

This post has been migrated from (A51.SE)

Your answer

Please use answers only to (at least partly) answer questions. To comment, discuss, or ask for clarification, leave a comment instead.
To mask links under text, please type your text, highlight it, and click the "link" button. You can then enter your link URL.
Please consult the FAQ for as to how to format your post.
This is the answer box; if you want to write a comment instead, please use the 'add comment' button.
Live preview (may slow down editor)   Preview
Your name to display (optional):
Privacy: Your email address will only be used for sending these notifications.
Anti-spam verification:
If you are a human please identify the position of the character covered by the symbol $\varnothing$ in the following word:
$\varnothing\hbar$ysicsOverflow
Then drag the red bullet below over the corresponding character of our banner. When you drop it there, the bullet changes to green (on slow internet connections after a few seconds).
Please complete the anti-spam verification




user contributions licensed under cc by-sa 3.0 with attribution required

Your rights
...